www.vorhilfe.de
- Förderverein -
Der Förderverein.

Gemeinnütziger Verein zur Finanzierung des Projekts Vorhilfe.de.
Hallo Gast!einloggen | registrieren ]
Startseite · Mitglieder · Impressum
Forenbaum
^ Forenbaum
Status VH e.V.
  Status Vereinsforum

Gezeigt werden alle Foren bis zur Tiefe 2

Navigation
 Startseite...
 Suchen
 Impressum
Das Projekt
Server und Internetanbindung werden durch Spenden finanziert.
Organisiert wird das Projekt von unserem Koordinatorenteam.
Hunderte Mitglieder helfen ehrenamtlich in unseren moderierten Foren.
Anbieter der Seite ist der gemeinnützige Verein "Vorhilfe.de e.V.".
Partnerseiten
Weitere Fächer:

Open Source FunktionenplotterFunkyPlot: Kostenloser und quelloffener Funktionenplotter für Linux und andere Betriebssysteme
Forum "Lineare Algebra - Moduln und Vektorräume" - Direkte Summe, UVR, Induktion
Direkte Summe, UVR, Induktion < Moduln/Vektorraum < Lineare Algebra < Hochschule < Mathe < Vorhilfe
Ansicht: [ geschachtelt ] | ^ Forum "Lineare Algebra - Moduln und Vektorräume"  | ^^ Alle Foren  | ^ Forenbaum  | Materialien

Direkte Summe, UVR, Induktion: Frage (beantwortet)
Status: (Frage) beantwortet Status 
Datum: 23:55 Mo 28.11.2011
Autor: davux

Aufgabe
Es sei $K$ ein Körper. Wie aus der Vorlesung bekannt heißt ein $K$-Vektorraum $V$ direkte Summe zweier Unterräume [mm] $V_1$ [/mm] und [mm] $V_2$, [/mm] wenn $V = [mm] V_1 [/mm] + [mm] V_2$ [/mm] und [mm] $V_1 \cap V_2 [/mm] = [mm] \{0\}$. [/mm] Man schreibt dann $V = [mm] V_1 \oplus V_2$. [/mm] Ebenso definiert man induktiv: $V$ heißt direkte Summe von Unterräumen [mm] $V_1, [/mm] ... , [mm] V_n$, [/mm] wenn $V = [mm] (V_1 \oplus [/mm] ... [mm] \oplus V_{n-1}) \oplus V_n$. [/mm] Beweisen oder widerlegen Sie folgende Aussagen:

(a) $V$ ist genau dann direkte Summe von [mm] $V_1, [/mm] ..., [mm] V_n$, [/mm] wenn $V = [mm] V_1 [/mm] + ... + [mm] V_n$ [/mm] und [mm] $V_i \cap \sum_{j\not=i} V_j [/mm] = [mm] \{0\}$ [/mm] für alle $i=1, ... , n$.

(b) $V$ ist genau dann direkte Summe von [mm] $V_1 [/mm] , ... , [mm] V_n$, [/mm] wenn [mm] $V=V_1 [/mm] + ... + [mm] V_n$ [/mm] und [mm] $V_i\cap V_j=\{0\}$ [/mm] für alle [mm] $i\not=j$. [/mm]

Hinweis: Zum Nachweis einer oder gegebenenfalls beider Aussagen ist die Verwendung eines Induktionsbeweises sinnvoll.

Hallo,

die Woche ist noch jung und ich habe noch viel Zeit über diese Aufgabe zu brüten. Bisher sehen für mich die Aufgabenteile (a) und (b) sich noch ziemlich ähnlich, aber irgendwas sagt mir, dass an einem etwas faul ist und ich ihn widerlegen müsste oder Einschränkungen angeben sollte, unter denen der Beweis doch gilt, sollte ich zu einem gelangen.
Ich hatte mir ein paar Beispiele angeschaut, wo aber zumeist von Basen oder Dimension die Rede war. Daher könnte ich mir vorstellen es argumentativ darauf zurückzuführen. Aber erstmal wollte ich gerne mal einen Versuch zu (a) kommentieren, korrigieren lassen.

direkte Summe von [mm] $V_1, [/mm] ..., [mm] V_n\hat=V_1\oplus [/mm] ... [mm] \oplus V_n=\oplus_{l=1}^n V_l=V$ [/mm]
[mm] $V=V_1 [/mm] + ... + [mm] V_n=\sum_{k=1}^n V_k$ [/mm]

Das wären die Vorüberlegungen gewesen. Irgendwie wollte ich auf ein sehr gewohntes Bild kommen um zu einem Induktionsbeweis zu gelangen. Ich nehme das folgende schon jetzt eher mit Humor, als dass ich wirklich daran glaube, aber so wirklich erklären, was daran falsch ist, kann ich mir auch nicht.

Beh.: Mit [mm] (\*:) $V_i \cap \sum_{j\not=i} V_j [/mm] = [mm] \{0\}$ [/mm]
gilt [mm] $\sum_{k=1}^n V_k [/mm] = [mm] \oplus_{l=1}^n$ [/mm]

Bew.:
IA, $n=2$:
[mm] $\sum_{k=1}^2 V_k [/mm] = [mm] V_1 [/mm] + [mm] V_2$ [/mm] mit [mm] $V_i \cap \sum_{j\not=i} V_j [/mm] = [mm] \{0\}$ [/mm]
[mm] $\gdw \oplus_{l=1}^2 V_l [/mm] = [mm] V_1 \oplus V_2$ [/mm]

IS, $n=n+1$:
[mm] $\sum_{k=1}^{n+1} V_k [/mm] = [mm] \sum_{k=1}^{n} (V_k) [/mm] + [mm] V_{n+1}$ [/mm]

$=^{IV} [mm] \oplus_{l=1}^n (V_l) [/mm] + [mm] V_{n+1}$ [/mm]

$=^{Mit [mm] (\*)} \oplus_{l=1}^n (V_l) \oplus V_{n+1}$ [/mm]

$= [mm] \oplus_{l=1}^{n+1} V_l$. [/mm]

        
Bezug
Direkte Summe, UVR, Induktion: Antwort
Status: (Antwort) fertig Status 
Datum: 10:59 Di 29.11.2011
Autor: Schadowmaster

moin davux,

Deine Induktion sieht garnicht mal sooo schlecht aus.
In der vorletzten Zeile, wo du * benutzt, solltest du vielleicht nochmal ganz detailliert ausführen wieso das aus * folgt.
Noch etwas schöner wird es, wenn du die Definition der direkten Summe nimmst.
Also mal an einem Beispiel:
$V = [mm] (V_1 [/mm] + [mm] V_2) [/mm] + [mm] V_3$ [/mm] ist genau dann eine direkte Summe, wenn gilt:
$1. [mm] V_1 \cap V_2 [/mm] = [mm] \{0\}$ [/mm]
$2. [mm] (V_1 [/mm] + [mm] V_2) \cap V_3 [/mm] = [mm] \{0\}$ [/mm]

Entsprechend hättest du das für:
$V = [mm] ((V_1 [/mm] + [mm] V_2) [/mm] + [mm] V_3) [/mm] + [mm] V_4$ [/mm]

Also denke dir mal die ganzen Klammern in die Gleichung rein.
Hier hast du nun links vom + und rechts jeweils nur einen Vektorraum stehen (der auf der linken Seite ist halt nur etwas kompliziert geschrieben, aber was solls?^^) und kannst die Definition für eine direkte Summe wie in meinem Beispiel drauf anwenden.

Das ist im Endeffekt das, was mit "man definiert induktiv: ..." gemeint ist.

Wenn du diese Definition verinnerlichst und nochmal ganz formal in deiner Induktion auftauchen lässt so taugt der Beweis durchaus was.

Und du hast Recht, bei der b) ist was faul, guck dir auch dafür die Definition und mein Beispiel an und überleg dir was da schiefgehen kann.
Als Tipp:
Nimm dir den [mm] $\IR^2$ [/mm] und geeignete Unterräume, da findet man schon ganz schöne Gegenbeispiele.


lg

Schadow

Bezug
                
Bezug
Direkte Summe, UVR, Induktion: Frage (beantwortet)
Status: (Frage) beantwortet Status 
Datum: 21:27 Mi 30.11.2011
Autor: yangwar1

Wie würde denn ein Beispiel aussehen?

Bezug
                        
Bezug
Direkte Summe, UVR, Induktion: Antwort
Status: (Antwort) fertig Status 
Datum: 21:44 Mi 30.11.2011
Autor: Schadowmaster

moin yangwar,

Überleg dir mal folgendes:
Ist V die direkte Summe von [mm] $V_1,V_2,\cdots,V_n$ [/mm] so ist dim(V) = [mm] dim($V_1$)+dim($V_2$)+\cdots +dim($V_n$) [/mm]
Wenn du also den [mm] $\IR^2$ [/mm] als direkte Summe schreiben möchtest brauchst du dafür zwei Unterräume der Dimension 1.
Findest du drei Unterräume, die alle Dimension 1 haben und die paarweise nur die 0 im Schnitt haben?
Als Tipp: Zeichne dir mal ein Koordinatensystem für den [mm] $\IR^2$ [/mm] und überlege dir, was genau eindimensionale Unterräume in dem Zusammenhang sind.

lg

Schadow

Bezug
                                
Bezug
Direkte Summe, UVR, Induktion: Mitteilung
Status: (Mitteilung) Reaktion unnötig Status 
Datum: 22:13 Mi 30.11.2011
Autor: yangwar1

Meintest du 2 oder 3 Untervektorräume?
Im $ [mm] \IR^2 [/mm] $ wäre das zum einen die x-Achse und zum anderen die y-Achse. Der Schnitt wäre 0.

Bezug
                                        
Bezug
Direkte Summe, UVR, Induktion: Frage (beantwortet)
Status: (Frage) beantwortet Status 
Datum: 14:12 Do 01.12.2011
Autor: yangwar1

Ist das so richtig?


Bezug
                                                
Bezug
Direkte Summe, UVR, Induktion: Antwort
Status: (Antwort) fertig Status 
Datum: 13:11 Fr 02.12.2011
Autor: angela.h.b.


> Ist das so richtig?

Hallo,

ich hoffe, daß ich das treffe, was Du mit "das" meinst.

Richtig ist, daß die x- und y-Achse Unterräume des [mm] \IR^2 [/mm] sind und daß ihr Schnitt nur aus der Null besteht.

Nicht richtig ist - falls Du dies denkst - daß diese beiden die einzigen eindimensionalen Unterräume des [mm] \IR^2 [/mm] sind. Da gibt es mehr.

Gruß v. Angela


>  


Bezug
                                                        
Bezug
Direkte Summe, UVR, Induktion: Frage (beantwortet)
Status: (Frage) beantwortet Status 
Datum: 13:23 Fr 02.12.2011
Autor: yangwar1

Ich meinte schon ersteres.

Also muss ich praktisch 3 eindimensionale Untervektorräue, sodass
V1+V2+V3=V gilt, der Schnitt aber nicht null ist?

Frage: Der Schnitt von was?
von V1 und V2 sowie von V2 und V3

Bezug
                                                                
Bezug
Direkte Summe, UVR, Induktion: Antwort
Status: (Antwort) fertig Status 
Datum: 13:39 Fr 02.12.2011
Autor: angela.h.b.


> Also muss ich praktisch 3 eindimensionale Untervektorräue,
> sodass
>  V1+V2+V3=V gilt, der Schnitt aber nicht null ist?

Hallo,

erstens mal verstehe ich Deinen Satz so rein von der Grammtik her überhaupt nicht.

Zweitens - entschuldige, ich bin schön etwas älter und weder geistig noch körperlich besonders beweglich - ist mir jetzt nicht ganz klar, welche Aufgabenstellung Du gerade bearbeitetst.
Die vom Schadowmaster?
Detektivarbeit sagt mir: hier ging es um b).
Er riet Dir zu versuchen, drei eindimensionale Unterräume des [mm] \IR^2 [/mm] zu suchen, deren Summe der [mm] \IR^2 [/mm] ist.
Diese Summe ist nicht direkt...

>  
> Frage: Der Schnitt von was?
>  von V1 und V2 sowie von V2 und V3

...obgleich Du die Unterräume so finden kannst, daß die 3 (!) paarweisen Schnitte jeweils nur den Nullvektor enthalten.
Mit diesem Gegenbeispiel hast Du dann b) widerlegt.

Gruß v. Angela


Bezug
        
Bezug
Direkte Summe, UVR, Induktion: Beweis
Status: (Frage) beantwortet Status 
Datum: 20:05 Mi 30.11.2011
Autor: yangwar1

Aufgabe
Es sei $ K $ ein Körper. Wie aus der Vorlesung bekannt heißt ein $ K $-Vektorraum $ V $ direkte Summe zweier Unterräume $ [mm] V_1 [/mm] $ und $ [mm] V_2 [/mm] $, wenn $ V = [mm] V_1 [/mm] + [mm] V_2 [/mm] $ und $ [mm] V_1 \cap V_2 [/mm] = [mm] \{0\} [/mm] $. Man schreibt dann $ V = [mm] V_1 \oplus V_2 [/mm] $. Ebenso definiert man induktiv: $ V $ heißt direkte Summe von Unterräumen $ [mm] V_1, [/mm] ... , [mm] V_n [/mm] $, wenn $ V = [mm] (V_1 \oplus [/mm] ... [mm] \oplus V_{n-1}) \oplus V_n [/mm] $. Beweisen oder widerlegen Sie folgende Aussagen:

(a) $ V $ ist genau dann direkte Summe von $ [mm] V_1, [/mm] ..., [mm] V_n [/mm] $, wenn $ V = [mm] V_1 [/mm] + ... + [mm] V_n [/mm] $ und $ [mm] V_i \cap \sum_{j\not=i} V_j [/mm] = [mm] \{0\} [/mm] $ für alle $ i=1, ... , n $.

Hallo,

ich habe diese Aufgabe auf meinem Übungsblatt und möchte sie beweisen, habe allerdings Probleme damit.
Zu zeigen ist eine Aquivalenzrelation.
Also zuerst:
"=>" Sei V direkte Summe von V1,...,Vn. Dann gilt V=V1+...+Vn und $ [mm] V_i \cap \sum_{j\not=i} V_j [/mm] = [mm] \{0\} [/mm] $ nach der Definition der direkten Summe.

"<=" Bei der Rückrichtung nehme ich an: V=V1+...+Vn und $ [mm] V_i \cap \sum_{j\not=i} V_j [/mm] = [mm] \{0\} [/mm] $ für alle i=1,...,n.
V=V1+...+Vn. Dann gilt: V ist direkte Summe von 2 Untervektorräumen, wenn V1+V2=V. ...

Irgendwie fehlt mir der Ansatz.
Ich könnte mir vorstellen, dass man zum beispiel irgendwie so vorgeht:
V=V1+V2
V=(V1+V2)+V3
Also man sagt, V1+V2=V, dann ist auch der nächste UVR addiert mit den beiden ersten ein UVR.
Und dies muss dann fortgesetzt werden (bewiesen mit Induktion).
Ich bräuchte auf jeden Fall einmal eine Erklärung.

Ich habe diese Frage in keinem Forum auf anderen Internetseiten gestellt.

Bezug
                
Bezug
Direkte Summe, UVR, Induktion: Antwort
Status: (Antwort) fertig Status 
Datum: 20:47 Mi 30.11.2011
Autor: Marcel

Hallo,

> Es sei [mm]K[/mm] ein Körper. Wie aus der Vorlesung bekannt heißt
> ein [mm]K [/mm]-Vektorraum [mm]V[/mm] direkte Summe zweier Unterräume [mm]V_1[/mm]
> und [mm]V_2 [/mm], wenn [mm]V = V_1 + V_2[/mm] und [mm]V_1 \cap V_2 = \{0\} [/mm]. Man
> schreibt dann [mm]V = V_1 \oplus V_2 [/mm]. Ebenso definiert man
> induktiv: [mm]V[/mm] heißt direkte Summe von Unterräumen [mm]V_1, ... , V_n [/mm],
> wenn [mm]V = (V_1 \oplus ... \oplus V_{n-1}) \oplus V_n [/mm].
> Beweisen oder widerlegen Sie folgende Aussagen:
>  
> (a) [mm]V[/mm] ist genau dann direkte Summe von [mm]V_1, ..., V_n [/mm], wenn
> [mm]V = V_1 + ... + V_n[/mm] und [mm]V_i \cap \sum_{j\not=i} V_j = \{0\}[/mm]
> für alle [mm]i=1, ... , n [/mm].
>  Hallo,
>  
> ich habe diese Aufgabe auf meinem Übungsblatt und möchte
> sie beweisen, habe allerdings Probleme damit.
>  Zu zeigen ist eine Aquivalenzrelation.
>  Also zuerst:
>  "=>" Sei V direkte Summe von V1,...,Vn. Dann gilt
> V=V1+...+Vn und [mm]V_i \cap \sum_{j\not=i} V_j = \{0\}[/mm] nach
> der Definition der direkten Summe.

meines Erachtens ist das zu knapp. Per Definitionem ist mir das so nur für den Fall [mm] $i=n\,$ [/mm] klar (unter Beachtung der Kommutativität der Schnittbildung, $A [mm] \cap [/mm] B=B [mm] \cap [/mm] A$)! Dass für jede Wahl von [mm] $i\,$ [/mm] sicher [mm] $V=V_1+\ldots+V_n$ [/mm] ist, folgt natürlich schon direkt aus der Voraussetzung - wobei man auch hier sowas wie Kommutativität und Assoziativität der Addition bzgl. des Grundvektorraums, der die Unterräume enthält, erwähnen sollte.
Was aber dann noch zu zeigen bleibt:
Ist $i [mm] \in \{1,\ldots,n\}$ [/mm] beliebig und ist $x [mm] \in V_i \cap \sum_{j \not=i}V_j\,,$ [/mm] so muss zwingend [mm] $x=0\,$ [/mm] sein (siehe unten,[1]). Das folgt meines Erachtens nicht unmittelbar daraus, dass [mm] $V\,$ [/mm] als direkte Summe der [mm] $V_j$ [/mm] geschrieben werden kann.
  

> "<=" Bei der Rückrichtung nehme ich an: V=V1+...+Vn und
> [mm]V_i \cap \sum_{j\not=i} V_j = \{0\}[/mm] für alle i=1,...,n.
> V=V1+...+Vn. Dann gilt: V ist direkte Summe von 2
> Untervektorräumen, wenn V1+V2=V. ...
>  
> Irgendwie fehlt mir der Ansatz.

Ja, Du denkst hier viel zu kompliziert. Bei der Rückrichtung wird natürlich [mm] $$(I)\;\;V=V_1+\ldost+V_n$$ [/mm]
angenommen UND es gelte
[mm] $$V_i \cap \sum_{j\not=i} V_j [/mm] = [mm] \{0\}$$ [/mm]
für alle $i [mm] \in \{1,\ldots,n\}\,.$ [/mm] Wenn das doch für alle diese [mm] $i\,$ [/mm] gilt, dann gilt es insbesondere für den speziellen Fall, dass [mm] $i=n\,,$ [/mm] ist. Setzt Du also [mm] $i=n\,,$ [/mm] so steht da, dass die Bedingung
[mm] $$V_n \cap \sum_{j=1}^{n-1}V_j=\{0\}$$ [/mm]
ist. Wie oben erwähnt läßt sich das auch schreiben als
[mm] $$(II)\;\;\;(\sum_{j=1}^{n-1}V_j) \cap V_n=\{0\}\,.$$ [/mm]

Es gelten nun also [mm] $(I)\,$ [/mm] und [mm] $(II)\,,$ [/mm] und das ist gerade nichts anderes als eine Umformulierung (per Definitionem) von
[mm] $$V=(V_1 \oplus \ldots \oplus V_{n-1})\oplus V_n\,.$$ [/mm]
(Naja gut, das stimmt so wirklich nicht ganz. Aber Du kannst es algorithmisch klar aufschreiben, wie es zu verstehen ist.
[mm] $V'_1=V_1+V_2$ [/mm] ist unter der genannten Voraussetzung insbesondere eine direkte Summe (beachte [mm] $(V_2 \cap V_1) \subseteq V_2 \cap \sum_{j \not=2}V_j=\{0\} \Rightarrow V_1 \cap V_2=\{0\}$), $V'_2=V'_1+V_3=(V_1+V_2)+V_3$ [/mm] ist unter der genannten Voraussetzung direkte Summe (beachte [mm] $(V_3 \cap (V_1+V_2)) \subseteq V_3 \cap \sum_{j \not=3}V_j=\{0\} \Rightarrow (\underbrace{V_1+V_2}_{=V_1 \oplus V_2}) \cap V_3=\{0\}$) [/mm] etc. pp.)

Der Knackpunkt des Beweises ist also der oben angedeutete im Teil [mm] $\Rightarrow\,:$ [/mm]
Unter der Voraussetzung, dass [mm] $V\,$ [/mm] sich als die obenstehende direkte Summe schreiben läßt, muss für ein jedes beliebiges $i [mm] \in \{1,\ldots,n\}$ [/mm] gelten, dass aus
$$x [mm] \in (V_i \cap \sum_{j \not= i}V_j)$$ [/mm]
folgt, dass [mm] $x=0\,$ [/mm] ist.

[mm] $\text{(}$[1] [/mm] Beachte: Dass für jedes [mm] $i=1,\ldots,n$ [/mm] gilt, dass [mm] $\{0\}\subseteq (V_i \cap \sum_{j \not= i}V_j)$ [/mm] ist klar (Schnitte von Unterräumen enthalten zwangsläufig stets den Nullvektor), und die obige Folgerung, wenn Du sie denn beweisen kannst, besagt dann, dass auch [mm] $(V_i \cap \sum_{j \not= i}V_j) \subseteq \{0\}$ [/mm] gilt. Zusammen also
[mm] $$\{0\}= V_i \cap \sum_{j \not= i}V_j\,.\text{)}$$ [/mm]

Grüße,
Marcel

Bezug
                        
Bezug
Direkte Summe, UVR, Induktion: Frage (beantwortet)
Status: (Frage) beantwortet Status 
Datum: 21:47 Mi 30.11.2011
Autor: yangwar1

Ich komme mit deiner Antwort nciht ganz zurecht. Am besten beschreibe ich einmal was ich verstanden habe:

Wir haben in der Vorlesung definiert, dass:
Sei V ein K-Vektorraum, V1, V2 Untervektorräume von V.
V heißt direkte Summe wenn die beiden Bedingungen gelten: V=V1+V2
und V1 geschnitten V2 = {0}.

Die Aufgabe verlangt nun von mir eine Äquivalenzrelation zu zeigen:
=>Zuerst nehme ich also an, dass V die direkte Summe ist von (V1,...,Vn).
Nach Definition müssen 2 Bedingungen gelten, damit V direkte Summe von (V1,...,Vn) ist. Zum einen muss V1+...Vn=V sein und $ [mm] V_i \cap \sum_{j\not=i} V_j [/mm] = [mm] \{0\} [/mm] $ gelten.
Wo genau liegt jetzt da der Knackpunkt?
Zur anderen Richtung:
<=: Man nehme also an:     $ [mm] (I)\;\;V=V_1+\ldost+V_n [/mm] $
und     $ [mm] V_i \cap \sum_{j\not=i} V_j [/mm] = [mm] \{0\} [/mm] für $ alle $ i [mm] \in\{1,\ldots,n\}\,. [/mm] $ Da dies für alle i gilt, gilt es auch für n. Soweit habe ich hier deine Lösung verstanden. Warum folgt dann aber, n-1


Bezug
                                
Bezug
Direkte Summe, UVR, Induktion: Antwort
Status: (Antwort) fertig Status 
Datum: 23:14 Mi 30.11.2011
Autor: Marcel

Hallo,

> Ich komme mit deiner Antwort nciht ganz zurecht.

kann passieren. Dann gucken wir nochmal!

> Am besten
> beschreibe ich einmal was ich verstanden habe:
>  
> Wir haben in der Vorlesung definiert, dass:
>  Sei V ein K-Vektorraum, V1, V2 Untervektorräume von V.
> V heißt direkte Summe wenn die beiden Bedingungen gelten:
> V=V1+V2
>  und V1 geschnitten V2 = {0}.
>  
> Die Aufgabe verlangt nun von mir eine Äquivalenzrelation
> zu zeigen:

Das ist eine irreführende Wortwahl. Äquivalenzrelationen sind wieder was eigenes in der Mathematik. Du willst die Äquivalenz bzw. Gleichwertigkeit zweier Aussagen beweisen!

>  =>Zuerst nehme ich also an, dass V die direkte Summe ist
> von (V1,...,Vn).
>  Nach Definition müssen 2 Bedingungen gelten, damit V
> direkte Summe von (V1,...,Vn) ist. Zum einen muss
> V1+...Vn=V sein und [mm]V_i \cap \sum_{j\not=i} V_j = \{0\}[/mm]
> gelten.
> Wo genau liegt jetzt da der Knackpunkt?

Nein. Jetzt verwechselst Du, was die Voraussetzung ist mit dem, was zu zeigen ist. Du setzt voraus, dass
[mm] $$V=V_1 \oplus \ldots \oplus V_n\,,$$ [/mm]
und das bedeutet, dass zum einen
[mm] $$V=V_1 [/mm] + [mm] \ldots [/mm] + [mm] V_n\,,$$ [/mm]
und (nach der rekursiven Definition), dass zudem
[mm] $$(V_1 \oplus \ldots \oplus V_{n-1}) \cap V_n=\{0\}\,.$$ [/mm]
Das gilt nach Voraussetzung.

Behauptet wird nun, dass, für ein jedes, beliebige $i [mm] \in \{1,\ldots,n\}\,$ [/mm] dann gelten:
[mm] $$(I)\;\;\;V=(\sum_{j \not=i}V_j)+V_i$$ [/mm]
und
[mm] $$(II)\;\;\;V_i \cap (\sum_{j \not=i}V_j)=\{0\}\,.$$ [/mm]

Die Summe in [mm] $(I)\,$ [/mm] ist wohldefiniert und [mm] $=V_1+\ldots+V_n\,,$ [/mm] unabhängig von der Wahl von [mm] $i\,,$ [/mm] weil die so charakterisierten Vektoren Elemente eines betrachteten "Obervektorraums" (einer, der eben alle Unterräume [mm] $V_i$ [/mm] enthält) sind, der die Assoziativität und Kommutativität der Addition gewährleisten muss - sonst wäre es kein Vektorraum. Die Bedingung [mm] $(II)\,$ [/mm] ist aber keineswegs klar. Sie ist nach Voraussetzung sicher für den Fall [mm] $i=n\,$ [/mm] klar, aber für jedes $i [mm] \in \{1,\ldots, n-1\}$ [/mm] muss sie nachgewiesen werden. Hier braucht man aber keine Einschränkung beim Beweis. Du nimmst nun [mm] $i\,$ [/mm] und setzt nur die Eigenschaft $i [mm] \in \{1,\ldots,n\}$ [/mm] voraus. Dann zeigst Du, dass gilt:
$$x [mm] \in V_i \cap \sum_{j \not=i}V_j \Rightarrow x=0\,.$$ [/mm]
Warum das reicht, um [mm] $V_i \cap \sum_{j \not=i}V_j=\{0\}$ [/mm] einzusehen, habe ich oben erläutert. Die Beliebigkeit von $i [mm] \in \{1,\ldots,n\}$ [/mm] zeigt dann, dass für jedes $i [mm] \in \{1,\ldots,n\}$ [/mm] gilt:
[mm] $$V_i \cap \sum_{j \not=i}V_j=\{0\}\,.$$ [/mm]
Der Knackpunkt hier ist also der Beweis, dass [mm] $(II)\,$ [/mm] für JEDES $i [mm] \in \{1,\ldots,n\}$ [/mm] gilt!

>  Zur anderen Richtung:
>  <=: Man nehme also an:     [mm](I)\;\;V=V_1+\ldots+V_n[/mm]
>  und     [mm]V_i \cap \sum_{j\not=i} V_j = \{0\} [/mm] für alle [mm]i \in\{1,\ldots,n\}\,.[/mm]
> Da dies für alle i gilt, gilt es auch für n. Soweit habe
> ich hier deine Lösung verstanden. Warum folgt dann aber,
> n-1

  
Okay, ich habe es verwirrend aufgeschrieben. Machen wir es mal ganz sauber:
1.) Die Summe [mm] $V'_1=V_1+V_2$ [/mm] ist direkt. Dazu müssen wir [mm] $V_1 \cap V_2=\{0\}$ [/mm] zeigen. Weil aber [mm] $V_1 \subseteq \sum_{j \not=2} V_j$ [/mm] und nach Voraussetzung (für [mm] $i=2\,$) [/mm] gilt [mm] $V_2 \cap \sum_{j \not=2} V_j=\{0\}$ [/mm] ist, muss auch
[mm] $$(V_1 \cap V_2)=(V_2 \cap V_1) \subseteq (V_2 \cap \sum_{j \not=2}V_j)$$ [/mm]
und damit
[mm] $$(V_2 \cap V_1) \subseteq \{0\}$$ [/mm]
gelten. Weil [mm] $\{0\} \subseteq (V_1 \cap V_2)$ [/mm] klar ist (Schnitte von Unterräumen enthalten stets den Nullvektor), folgt
[mm] $$V_1 \cap V_2=\{0\}\,.$$ [/mm]

2.) Wegen 1.) ist die Summe [mm] $V'_2=V'1+V_3$ [/mm] direkt:
Es ist ja [mm] $V'_2=V'_1+V_3=(V_1+V_2)+V_3=V_1+V_2+V_3$ [/mm] und zu zeigen ist nun nur noch, dass
$$V'_1 [mm] \cap V_3=\{0\}$$ [/mm]
respektive
[mm] $$V_3 \cap V'_1=\{0\}$$ [/mm]
gilt.

Analog zu 1.) beachtest Du nun [mm] $V'_1=V_1+V_2 \subseteq \sum_{j \not=3}V_j$ [/mm] und dass nach Voraussetzung [mm] $V_3 \cap \sum_{j \not=3}V_j=\{0\}$ [/mm] ist, um dies einzusehen.

So fortfahrend erhälst Du dann den Schritt

(n-2).) Die Summe [mm] $V'_{n-2}=V'_{n-3}+V_{n-1}$ [/mm] ist direkt

und damit gelangst Du schließlich zu dem Schritt

(n-1).) Die Summe [mm] $V'_{n-1}=V'_{n-2}+V_n$ [/mm] ist direkt, also gilt
$$V'_{n-1}=V'_{n-2} [mm] \oplus V_n=(V_1 \oplus \ldots \oplus V_{n-1}) \oplus V_n\,.$$ [/mm]

Damit bist Du aber fertig, weil dann $V'_{n-1}$ gerade nichts anderes ist als [mm] $V_1+\ldots+V_n=V\,.$ [/mm] Es gilt also [mm] $V=V_1+\ldots+V_n=V_1 \oplus \ldots \oplus V_n\,.$ [/mm]

Diese "algorithmische" Vorgehensweise benutzt dabei aber eigentlich immer nur das gleiche Schema, welches sich Schritt für Schritt ergibt:
Im $k-$ten Schritt betrachtet man [mm] $V'_{k-1}=V_1+\ldots+V_k$ [/mm] und zeigt, dass
[mm] $$V_{k} \cap \sum_{j=1}^{k-1} V_k =\{0\}$$ [/mm]
ist. (Das sieht dann "eigentlich genauso aus wie die Voraussetzung für [mm] $i=n\,,$ [/mm] nur dass die Rolle des [mm] $n\,$'s [/mm] hier von einem [mm] $k\,$ [/mm] übernommen wird". Das meinte ich in obiger Antwort.)
Das ergibt sich, weil [mm] $(V_k \cap \sum_{j=1}^{k-1} V_k) \subseteq (V_k \cap \sum\limits_{\substack{j=1\\j\not=k}}^nV_j)=\{0\}$ [/mm] nach Voraussetzung gilt.


------
Mögliche Zusammenfassung bzw., wenn man das ganze ein wenig sauberer strukturieren will:
Man kann den Beweis auch so führen, indem man erst folgendes beweist:
Behauptung:
Falls [mm] $V_i \cap \sum_{j \not=i} V_j=\{0\}$ [/mm] für jedes $i [mm] \in \{1,\ldots,n\}$ [/mm] gilt, dann ist auch für jedes $1 [mm] \le [/mm] k [mm] \le [/mm] n-1$ die Summe
[mm] $$V'_k:=V_1+\ldots+V_k+V_{k+1}$$ [/mm]
direkt.

Beweis:
Dies folgt im Wesentlichen wegen [mm] $V'_k=V'_{k-1}+V_{k+1}$ [/mm] und aus
[mm] $$V'_{k-1}\cap V_{k+1}=(V_{k+1} \cap [/mm] V'_{k-1}) [mm] \subseteq (V_{k+1} \cap \sum_{j \not=k+1}V_j)=\{0\}\,,$$ [/mm]
wobei (für jedes betrachtete [mm] $k\,$) [/mm] die letzte Gleichung aus der Voraussetzung (für den Fall $i=k+1$) folgt.

Aus diesem "Hilfssatz" ergibt sich dann:
Für $k=n-1$ erhält man folglich, dass unter der Voraussetzung

    [mm] $V_i \cap \sum_{j \not=i} V_j=\{0\}$ [/mm] für jedes $i [mm] \in \{1,\ldots,n\}$ [/mm]

die Summe [mm] $V_1+\ldots+V_n$ [/mm] direkt ist.
Wegen [mm] $V=V_1+\ldots+V_n$ [/mm] also die Behauptung in der Aufgabe!

Gruß,
Marcel

Bezug
                
Bezug
Direkte Summe, UVR, Induktion: Mitteilung
Status: (Mitteilung) Reaktion unnötig Status 
Datum: 22:01 Mi 30.11.2011
Autor: Schadowmaster

Da du den Thread hier ja auch schon entdeckt hast knote ich die beiden mal zusammen.

Bezug
                        
Bezug
Direkte Summe, UVR, Induktion: Mitteilung
Status: (Mitteilung) Reaktion unnötig Status 
Datum: 10:17 Do 01.12.2011
Autor: davux

Nett. :)
Bei Marcels antworten kann man sich ja richtig einen anlesen!

Bezug
        
Bezug
Direkte Summe, UVR, Induktion: Frage (beantwortet)
Status: (Frage) beantwortet Status 
Datum: 17:44 Do 01.12.2011
Autor: Fincayra

Huhu

Mir will es nicht einleuchten, wo DER Unterschied zwischen [mm] V_i \cap \summe V_j [/mm] und [mm] V_i \cap V_j [/mm] ist. Warum ist der Schnitt ein anderer wenn ich mehrere [mm] V_j [/mm] nehme, anstatt nur einem? Die 0 muss doch in einem drin sein, wenn sie auch in mehreren drin ist? *auf dem Schlauch steht*

LG
Fin

Bezug
                
Bezug
Direkte Summe, UVR, Induktion: Antwort
Status: (Antwort) fertig Status 
Datum: 18:10 Do 01.12.2011
Autor: Marcel

Hallo,

> Huhu
>  
> Mir will es nicht einleuchten, wo DER Unterschied zwischen
> [mm]V_i \cap \summe V_j[/mm] und [mm]V_i \cap V_j[/mm] ist.

warum sollte das das gleiche sein? (Im letzten Summenzeichen sollte das [mm] $j\,$ [/mm] dann sicher auch nicht [mm] $=i\,$ [/mm] werden können und zudem alle verbleibenden Werte von [mm] $\{1,\ldots,n\}$ [/mm] durchlaufen).
Natürlich gilt in beiden Fällen, dass [mm] $\{0\}$ [/mm] eine Teilmenge des jeweiligen Schnitts ist. Aber ansonsten stehen doch da i.a. ganz unterschiedliche Mengen:
Z.B. anschaulisch:
Wenn [mm] $V_1$ [/mm] "die [mm] $x\,$-Achse [/mm] des [mm] $\IR^2$" [/mm] und [mm] $V_2$ [/mm] "die [mm] $y\,$-Achse" [/mm] des [mm] $\IR^2$ [/mm] ist, dann ist [mm] $V_1 \cap V_2=\{(0,0)^T \in \IR^2\}\,.$ [/mm]
Nimmst Du nun [mm] $V_3$ [/mm] als "Winkelhalbierende" des [mm] $\IR^2\,,$ [/mm] so ist die (direkte) Summe [mm] $V_2+V_3$ [/mm] der ganze [mm] $\IR^2$ [/mm] und damit
[mm] $$V_1 \cap (V_2+V_3)=V_1\,,$$ [/mm]
und da "die [mm] $x\,$-Achse" $V_1$ [/mm] nicht nur aus dem Nullpunkt des [mm] $\IR^2$ [/mm] besteht, ist damit
[mm] $$V_1 \cap V_2=\{0\} \subset V_1=V_1 \cap (V_2+V_3)\,,$$ [/mm]
wobei [mm] $\subset$ [/mm] hier im Sinne von "echter Teilmenge" zu verstehen ist.

Was Du immer sehen wirst:
[mm] $$\{0\}\subseteq V_i \cap V_j$$ [/mm]
für jede Wahl von paarweise verschiedenen [mm] $i\,$ [/mm] und [mm] $j\,,$ [/mm] sofern es sich bei [mm] $V_i$ [/mm] und [mm] $V_j$ [/mm] denn um "Unterräume" handelt und damit auch
[mm] $$\{0\} \subseteq V_i \cap V_j \subseteq V_i \cap \sum_{k \not=i}V_k\,.$$ [/mm]

Das letzte [mm] $\subseteq$ [/mm] gilt,  weil zum einen für $j [mm] \not=i$ [/mm] dann [mm] $V_j \subseteq \sum_{k\not=i}V_k$ [/mm] gilt, und zum anderen, weil aus $B [mm] \subseteq [/mm] B'$ sofort
$$(A [mm] \cap [/mm] B) [mm] \subseteq [/mm] (A [mm] \cap [/mm] B')$$
folgt. (Beweis?)

Gruß,
Marcel

Bezug
                        
Bezug
Direkte Summe, UVR, Induktion: Frage (beantwortet)
Status: (Frage) beantwortet Status 
Datum: 19:17 Do 01.12.2011
Autor: Fincayra

Hi

Mh, ich glaube zu verstehen, was du geschrieben hast. Mir leuchtet das im Zusammenhang mit der Aufgabe aber noch nicht ein.
Die Aussage von a) ist ja richtig und die Aussage von b) falsch.
Ist das:

$ [mm] V_1 \cap (V_2+V_3)=V_1\,, [/mm] $

nicht das, was die a) sagt? Dann wäre doch a) falsch? Es soll doch nur 0 im Schnitt haben.

Bezug
                                
Bezug
Direkte Summe, UVR, Induktion: Antwort
Status: (Antwort) fertig Status 
Datum: 00:20 Fr 02.12.2011
Autor: Marcel

Hallo,

> Hi
>  
> Mh, ich glaube zu verstehen, was du geschrieben hast. Mir
> leuchtet das im Zusammenhang mit der Aufgabe aber noch
> nicht ein.
>  Die Aussage von a) ist ja richtig und die Aussage von b)
> falsch.

jetzt habe ich erst gesehen, dass oben eine ursprüngliche Aufgabenstellung steht. Ich hatte oben eigentlich auf eine andere, direkte Frage geantwortet, und die Threads wurden zusammengefügt. Nur, falls der ein oder andere mal verwirrt ist.

> Ist das:
>  
> [mm]V_1 \cap (V_2+V_3)=V_1\,,[/mm]
>  
> nicht das, was die a) sagt? Dann wäre doch a) falsch? Es
> soll doch nur 0 im Schnitt haben.

Nein, die Aussage aus a) ist nicht falsch. Man kann beweisen, dass a) gilt, dass nämlich jede der beiden Aussagen (Folgerungen!)

1.) Wenn [mm] $V\,$ [/mm] direkte Summe von [mm] $V_1,\ldots,V_n$ [/mm] ist, dann gelten [mm] $V=V_1+\ldots+V_n$ [/mm] UND [mm] $V_i \cap \sum_{j \not=i}V_j=\{0\}$ [/mm] für alle [mm] $i=1,\ldots,n$ [/mm]

und

2.) Wenn [mm] $V=V_1+\ldots+V_n$ [/mm] UND [mm] $V_i \cap \sum_{j \not=i}V_j=\{0\}$ [/mm] für alle [mm] $i=1,\ldots,n$ [/mm] gelten, dann ist [mm] $V\,$ [/mm] direkte Summe der [mm] $V_1,\ldots,V_n$ [/mm]

richtig ist.

(Beachte: Wenn man Aussage [mm] $A\,$ [/mm] setzt als: [mm] "$V\,$ [/mm] direkte Summe von [mm] $V_1,\ldots,V_n\,,$" [/mm] und Aussage [mm] $B\,:$ "$V=V_1+\ldots+V_n$ [/mm] UND [mm] $V_i \cap \sum_{j \not=i}V_j=\{0\}$ [/mm] für alle [mm] $i=1,\ldots,n$", [/mm] dann steht in [mm] $A\,$ [/mm] nichts anderes als die Behauptung, dass [mm] $A\,$ [/mm] und [mm] $B\,$ [/mm] gleichwertig sind: $A [mm] \gdw B\,.$ [/mm] Wenn man dies beweisen will, dann hat man zwei Folgerungen, nämlich $A [mm] \Rightarrow [/mm] B$ UND $B [mm] \Rightarrow [/mm] A$ zu beweisen!)

Gehe nun mal davon aus, dass $A [mm] \gdw [/mm] B$ gilt.

Dass oben nun [mm] $V_1 \cap (V_2+V_3)=V_1 \not=\{0\}$ [/mm] ist, besagt dann, dass hier die Aussage [mm] $B\,$ [/mm] falsch sein muss. Weil $A [mm] \gdw B\,,$ [/mm] kann aber dann auch [mm] $A\,$ [/mm] nicht gelten. (Denn wäre [mm] $A\,$ [/mm] richtig, so würde aus $A [mm] \Rightarrow [/mm] B$ folgen, dass auch [mm] $B\,$ [/mm] richtig sein müsste. Widerspruch. (Alternative und meist bevorzugte Argumentation: Kontraposition; ist aber eigentlich auch die gleiche Argumentation...))
Weil [mm] $B\,$ [/mm] nicht gilt, aber $A [mm] \gdw [/mm] B$ erkannt wurde, kann also auch [mm] $A\,$ [/mm] nicht gelten. Daraus ergibt sich also, dass die obenstehende Summe [mm] $V:=V_1+(V_2+V_3)$ [/mm] dann nicht direkt sein kann. Mehr nicht!

P.S.:
Du musst genau drauf achten, dass, wenn Du $A [mm] \gdw [/mm] B$ beweisen sollst, nur die Richtigkeit der beiden Folgerungen
$$A [mm] \Rightarrow [/mm] B$$
und
$$B [mm] \Rightarrow [/mm] A$$
nachzuweisen hast. Es geht nicht darum, etwa zu prüfen, ob [mm] $A\,$ [/mm] gilt. Beim Beweis der Aussage
$$A [mm] \Rightarrow [/mm] B$$
setzt Du vielmehr voraus, dass [mm] $A\,$ [/mm] gelte und hast dann nachzuweisen, dass dann auch [mm] $B\,$ [/mm] gelten muss!

P.P.S.:
Ohne a) zu benutzen kann man mit obigem Beispiel auch direkt einsehen, dass b) falsch ist. Denn offensichtlich ist oben [mm] $V_1 \cap V_2=V_1 \cap V_3=V_2 \cap V_3=\{(0,0)^T \in \IR^2\}$ [/mm] und mit [mm] $V=V_1+V_2+V_3=\IR^2$ [/mm] kann dann aber [mm] $V\,$ [/mm] nicht direkte Summe der [mm] $V_1,V_2,V_3$ [/mm] sein:
[mm] $V'_1=V_1+V_2=\IR^2$ [/mm] ist zwar direkt, aber
[mm] $$\IR^2=(V_1+V_2)+V_3=V'_1+V_3$$ [/mm]
kann wegen
$$V'_1 [mm] \cap V_3=V_3\not=\{(0,0)^T\}$$ [/mm]
die "rekursive Definition" nicht erfüllen: [mm] $V'_1+V_3$ [/mm] kann also keine direkte Summe sein.

Gruß,
Marcel

Bezug
Ansicht: [ geschachtelt ] | ^ Forum "Lineare Algebra - Moduln und Vektorräume"  | ^^ Alle Foren  | ^ Forenbaum  | Materialien


^ Seitenanfang ^
ev.vorhilfe.de
[ Startseite | Mitglieder | Impressum ]